Đến nội dung

Karl Heinrich Marx nội dung

Có 54 mục bởi Karl Heinrich Marx (Tìm giới hạn từ 09-06-2020)



Sắp theo                Sắp xếp  

#584766 $ab-c$ ; $bc-a$ ; $ca-b$ đều là lũy thừa của 2

Đã gửi bởi Karl Heinrich Marx on 24-08-2015 - 22:31 trong Số học

Tìm mọi số nguyên dương $a,b,c$ thỏa mãn : $ab-c$ ; $bc-a$ ; $ca-b$ đều là lũy thừa của 2 . 

 

NX : Mình lấy lại bài số học của IMO 2015 vừa qua để các trao đổi kĩ càng hơn về việc phân tích hướng giải , chỉ ra đâu là mấu chốt của bài  ,.... chứ không nên đùng đùng xét một loạt các TH mà không chú thích tại sao :)) . Mong các bạn không nên dẫn link trong bài viết này để tiện theo dõi .  

Nhìn vào theo cảm nhận mang tính cá nhân của mình thì thứ làm cho nó có hữu hạn nghiệm có thể khai thác được là sự lệch bậc trong biểu thức, vì nó đối xứng nên ta ta có thể dựa vào $v_2$ của tích 2 số phải lớn hơn $v_2$ của một số, rõ ràng nếu xem 3 số là như nhau thì để tạo ra lũy thừa của một số nguyên tố, đây là một điểm ta cảm thấy bất cập, đó chính là thứ cần vin vào để lập luận. Để đảm bảo cho mục đích vừa nêu ta chọn $a$ là số có $v_2$ lớn nhất.

Khi đó ta chứng minh được nếu $v_2(a)>v_2(b)$ và $v_2(a)>v_2(c)$ là vô lí.

Thật vậy đặt $a=2^i.a_1,b=2^j.b_1,c=2^k.c_1$

Nếu $i>j,i>k$ thì từ $ab-c,ac-b$ đều là lũy thừa của $2$ ta suy ra $2^{i-k}.2^j.b_1-c_1=1$ và $2^{i-j}.2^k.c_1-b_1=1$

Suy ra $1 \ge 2.b_1-c_1$ và $1 \ge 2.c_1-b_1$. Cái này chỉ xảy ra khi $b_1=c_1=1,j=k=0,i=1$, nghiệm này không phù hợp.

Vậy trong 2 số $i,j,k$ có ít nhất 2 số bằng nhau. Ta giả sử là $i=j$ khi đó ta có:

$ca-b,cb-a$ là lũy thừa của 2.

_Nếu $c$ là số chẵn thì suy ra $ca_1-b_1=1,cb_1-a_1=1$, chú ý lúc này $v_2(c)>v_2(a_1),v_2(c)>v_2(b_1)$ nên theo trên ta suy ra nghiệm $c=2,a_1=b_1=1$

Thay vào để $ab-c$ là lũy thừa của $2$ thì suy ra $i=1$ nên $(2;2;2)$ là một nghiệm.

_Bây giờ xét $c$ là một số lẻ.

Ở đây sinh ra 2 vấn đề là nếu $a,b$ lẻ thì chưa nói được gì nhưng mà nếu $a,b$ chẵn thì ta có $ab-c=1$

+)Ta xét $a,b$ chẵn trước thì khi đó $c=2^{2i}a_1b_1-1$ (vì $i=j \ge 1$) và ta phải có $x=2^{2i}a_1^2b_1-a_1-b_1$ và $y=2^{2i}a_1b_1^2-a_1-b_1$ đều là các lũy thừa của 2. Nếu 2 số này bằng nhau thì $a_1=b_1$ thay vào có thể giải ra nghiệm $(2;2;3)$

Nếu 2 số này khác nhau thì $v_2(x) \ne v_2(y)$ nên $v_2(x+y)=v_2(x-y)$ suy ra $v_2(a_1+b_1)+1=v_2(a_1-b_1)+2i \Rightarrow v_2(a_1+b_1)=v_2(a_1-b_1)-1+2i \ge 2i$

Nếu $v_2(a_1+b_1)>2i$ thì ta có thể chứng minh được $x,y$ đều là lũy thừa của 2 là mâu thuẫn (cái này chắc không khó, nhưng mình nhác mò chứng minh :D)

Từ đó suy ra $v_2(a_1+b_1)=2i$ và $v_2(a_1-b_1)=1$

Đặt tiếp $a_1+b_1=2^i.r$ với $r$ lẻ và $z=a_1^2b_1-r,t=a_1b_1^2-r$ thì $z,t$ là lũy thừa của 2 mà $v_2(z-t)=v_2(a-b)=1$ suy ra một trong 2 số $z,t$ bằng 2.

Không mất tính tổng quát giả sử là $z=2$, nếu $a_1 \ge 2$ thì $a_1^2b_1-r \ge (a_1-1)a_1b_1+a_1b_1-r>2$ chú ý là $r \le \frac{1}{4}(a_1+b_1)$ nên $a_1b_1-r>0$.

Vậy $a_1=1$ suy ra $b_1=r+2$, ta có $2^{2i}r=a_1+b_1=r+3$ mà $i \ge 1$ nên suy ra luôn $r=1$. Từ đây suy ra được giá trị của $a,b,c$ ta được bộ nghiệm $(2,6,11)$

+) Bây giờ xét $a,b,c$ cùng lẻ

Thủ thuật cũng gần tương tự như trên một chút, nhưng trước tiên ta nhận xét là 3 số này không có 2 số nào bằng nhau và không có số nào bằng 1. Ngược lại nếu tồn tại thì dễ dàng chứng minh vô lí.

Với điều kiện trên không mất tính tổng quát $a>b>c$, khi đó ta có $ab-c>ac-b$, do đó $v_2(ac-b)=v_2[(ab-c)-(ac-b)]=v_2[(ab-c)+(ac-b)] $. Suy ra $v_2(ac-b)=v_2(a+1)+v_2(b-c)=v_2(a-1)+v_2(c+b)$, ta chú ý là trong 2 số $v_2(a-1)$ và $v_2(a+1)$ có một số bằng 1. Do đó hoặc là $v_2(b-c)=v_2(ac-b)-1$ hoặc là $v_2(b+c)=v_2(ac-b)-1$. Chung quy ta đều có $2(b+c) \ge ac-b$ vì $ac-b=2^{v_2(ac-b)}$, suy ra $3b \ge (a-2)c$, mà $a,b,c$ đều lẻ lớn hơn 1 và $a>b>c$ nên $a-2 \ge b, c \ge 3$. Đẳng thức trên phải xảy ra và xảy ra khi $a=b+2,c=3$

Ta có $ab-c=b(b+2)-3=(b-1)(b+3)$ là lũy thừa của 2 suy ra $b-1$ và $b+3$ đều là lũy thừa của 2 suy ra $b=5$.

Từ đây dẫn đến bộ nghiệm $(3,5,7)$

 

P/s: Mình không giỏi trình bày nên khá khó nhìn. Bài này không khó, và so với một bài số 5 trong kì thi IMO thì nó được xem là dễ.




#584795 CMR: tồn tại một chỉ số $i_o$ sao cho $a_{i_0}\...

Đã gửi bởi Karl Heinrich Marx on 25-08-2015 - 01:11 trong Số học

Bài toán: Cho tập $S=\{a_1, a_2, ..., a_n\}\subset \mathbb{N^*}$ và $P(x)\in\mathbb{Z}[x]$. Giả sử rằng với mọi $k\in\mathbb{Z^+}$ đều tồn tại chỉ số $i$ sao cho $a_i\;|\;P(k)$. CMR: tồn tại một chỉ số $i_o$ sao cho $a_{i_0}\;|\;P(k), k\in\mathbb{Z}$$

Trước tiên đặt $a$ là BCNN của tất cả các phần tử thuộc $S$.

Với mọi số $r \in Z$ thì luôn tồn tại số $k$ để $r+ka \in Z^+$, khi đó thì $P(r) \equiv P(r+ka)$ (mod $a$), mặt khác tồn tại $a_i$ để $a_i|P(r+ka)$ và $a_i|a$ ta suy ra $a_i|P(r)$.




#564899 Cho dãy $u_n=\sum_{k=0}^{2n} \frac{1...

Đã gửi bởi Karl Heinrich Marx on 11-06-2015 - 08:22 trong Dãy số - Giới hạn

Cho dãy số: $\qquad u_n=\sum_{k=0}^{2n} \frac{1}{n^2+k}\qquad (n=1,2,...)$

 

Tính tổng: $ \quad S_n=\sum_{k=1}^n \left\lfloor\frac{2}{u_k}\right\rfloor $

Bài toán mới nhìn vào có vẻ hơi đáng sợ nhưng bình tĩnh một chút đánh giá khách quan thì kiểu toán này chỉ có cách chặn 2 đầu thôi.

Ta có:

$$ \sum_{k=0}^{2n}\frac{1}{n^2}>u_n=\sum_{k=0}^{2n} \frac{1}{n^2+k}>\sum_{k=0}^{2n}\frac{1}{(n+1)^2} \Rightarrow \frac{2n+1}{n^2}>u_n>\frac{2n+1}{(n+1)^2}$$

Tuy nhiên đến đây mà chia 2 lật ngược lại thì chưa đạt được điều ta mong muốn, bây giờ ta cần một điều chặt hơn một chút là

$$ \frac{2n}{n^2}>u_n>\frac{2n+2}{(n+1)^2}$$

Tức là ta cần một tổng $i+1$ số hạng trong sigma biểu diễn $u_n$ có tổng bé hơn $\frac{i}{n^2}$ tức là $i+1$ số này đều không nhỏ hơn $\frac{i+1}{i}.n^2$, tự nhiên ta thử với $i=n$ thì đúng luôn chọn các số từ $n(n+1)$ đến $n(n+2)$. Như vậy ta viết lại $u_n$

$$ u_n=\sum_{k=0}^{n-1}\frac{1}{n^2+k}+\sum_{k=n}^{2n}\frac{1}{n^2+k}<n.\frac{1}{n^2}+(n+1).\frac{1}{n(n+1)}=\frac{2}{n}$$

$$ u_n=\sum_{k=0}^{n-1}\frac{1}{n^2+k}+\sum_{k=n}^{2n}\frac{1}{n^2+k}>n.\frac{1}{n(n+1)}+(n+1).\frac{1}{(n+1)^2}=\frac{2}{n+1}$$

Do vậy $n<\frac{2}{u_n}<n+1$

Suy ra $S_n=\frac{n(n+1)}{2}$




#566707 CMR $\exists a,b,c:c^2\mid a^2+b^2$

Đã gửi bởi Karl Heinrich Marx on 18-06-2015 - 18:37 trong Số học

em không có lời giải của bài toán cũng như không dám chắc về tính đúng đắn của nó

theo em thì cần một điều kiện gì đó của $n$ đủ chặt để bài toán đúng ,mong anh và mọi người góp ý

Khi thay $k= \frac{u}{v}$ thì ta có $\frac{a}{c}=\frac{u^2-v^2+2uv}{u^2+v^2}=1+\frac{2v(u-v)}{u^2+v^2},\frac{b}{c}=\frac{v^2-u^2+2uv}{u^2+v^2}=1+\frac{2u(v-u)}{u^2+v^2}$

Giờ nếu thay $u=n+2,v=n$ thì sẽ ra $\frac{a}{c}=\frac{n^2+4n+2}{n^2+2n+2},\frac{b}{c}=\frac{n^2-2}{n^2+2n+2}$

Trong ví dụ này nếu lấy đoạn $n^2;(n+2)^2$ cũng không được nhưng nếu lấy $n^2-2;(n+2)^2-2$ thì lại hợp lí. Tuy nhiên lời giải ta lại sử dụng đúng 2 mút thì nó lại mất hay vì thường người ta sẽ thử 2 mút trước ra ngay đáp án thì họ chả quan tâm phần làm sao tìm ra 2 mút này cả.

Có một cách tìm cận không chặt lắm nhưng mà tự nhiên hơn là thay số như ở trên.

Ta hình dung là $c$ sẽ nằm giữa 2 số $a,b$ và $|\frac{2v(u-v)}{u^2+v^2}|,|\frac{2u(v-u)}{u^2+v^2}|$ là các khảng cách từ $a,b$ đến $c$. Khoảng cách này càng bé càng tốt mà $u,v$ không thể bằng nhau nên ta cho $v=u+1$. 

Thay vào thì ta được $\frac{a}{c}=\frac{2v^2+4v+1}{2v^2+2v+1}, \frac{b}{c}=\frac{2v^2-1}{2v^2+2v+1}$

Chọn một cận là $n^2$ cận còn lại là $n^2+t$, trong 2 phân thức trên số ở mẫu nằm giữa 2 số ở tử nên chỉ cần chặn 2 số ở tử thôi.

tức là $2v^2-1 \ge n^2, 2(v+1)^2 \le n^2+t+1$

Tìm $t$ để thỏa mãn tồn tại số nguyên $v$. Cái này các e thử tiếp tục xem.

Việc đặt ra vấn đề chưa giải quyết đc và trao đổi với nhau như thế này là điều cần thiết khi tham gia diễn đàn, các e cố gắng phát huy nhé!




#566550 CMR $\exists a,b,c:c^2\mid a^2+b^2$

Đã gửi bởi Karl Heinrich Marx on 18-06-2015 - 05:16 trong Số học

$\boxed{\text{Problem}}$

Chứng minh rằng $\forall n\in \mathbb{N}^*$ thì giữa $n^2$ và $(n+1)^2$ luôn tồn tại ba số tự nhiên phân biệt $a,b,c$ sao cho

$c^2\mid a^2+b^2$

Bài này a cũng có một chút ý tưởng nhưng cũng loay hoay một lúc k ra, post lên để các e thử tiếp tục xem sao (dĩ nhiên là khi không ra thì vẫn hơi nghi ngờ tính đúng đắn của bài toán). Ta thử giải quyết với $n$ vô cùng lớn trước xem sao, đầu tiên ta thấy xét $\frac{a^2+b^2}{c^2}$ trong đó $n^2 \le a,b,c, \le (n+1)^2$ có thể suy ra là:

$$ \frac{2n^2}{(n+1)^2} \le \frac{a^2+b^2}{c^2} \le \frac{2(n+1)^2}{n^2}$$

Cho $n$ vô cùng lớn thì $\frac{a^2+b^2}{c^2} \rightarrow 2$.

Vậy với $n$ đủ lớn mà tồn tại $3$ số thỏa mãn đề bài thì ta phải có $a^2+b^2=2c^2 \Rightarrow (a-c)(a+c)=(c-b)(c+b)$

Đặt $(a-c)=k(c-b)$ khi đó $k \in Q$ và suy ra $b+c=k(a+c)$. Từ $2$ pt này có thể giải $a,b$ theo $k$ và $c$. Có vẻ như sẽ ra $a= \frac{1-k^2+2k}{k^2+1}.c,b=\frac{k^2-1+2k}{k^2+1}.c$. Đặt $k=\frac{u}{v}$ trong đó $u,v$ là các số nguyên, ta biểu diễn được $\frac{a}{c},\frac{b}{c}$ theo các phân thức với $u,v$. Bây giờ phải chứng minh tồn tại $u,v,$ để từ các phân thức đó nằm trong đoạn $(n^2,(n+1)^2$. Tuy nhiên chỗ này a thử cho một vài giá trị $u,v$ theo $n$ thì không ra. Chú ý một chút là vì giới hạn của $\frac{a}{c},\frac{b}{c}$ đều bằng $1$ khi $n$ về vô cùng nên khi biểu diễn $u,v$ theo $n$ nên chọn giá trị sao cho giới hạn của $\frac{u}{v}$ khi $n$ về vô cùng là $1$. Nếu e có lời giải bài này thì a rất muốn xem vì theo anh thì hình như nó không đúng với $n$ đủ lớn.




#675400 Một bài tổ hợp từ một bài số học

Đã gửi bởi Karl Heinrich Marx on 26-03-2017 - 20:42 trong Tổ hợp và rời rạc

Thể theo nguyện vọng của đồng chí supermember, lâu lâu lên diễn đàn trao đổi với đồng chí một tí, cũng hi vọng là có thể chia sẻ trao đổi chút gì đó với các bạn.

Từ một bài số học mình biết cũng khá lâu, hôm nay rỗi rãi phát biểu nó sang một bài tổ hợp. Cho một tập hợp $A$ gồm $n$ phần tử, và một dãy tập hợp con khác rỗng phân biệt của $A$ là $A_1,A_2,...,A_T$ với $T=(n-1)^2+1$. Với một dãy số nguyên dương tăng bất kì $a_1,a_2,..,a_n$ mà $a_n \le T$ thì tồn tại $i,j,1 \le i < j \le n$ và $A_{a_i}$ là một tập con của $A_{a_j}$. Chứng minh cái dãy tập hợp trên chứa A.

Thử giải xem nào đại ca supermember. Qua chủ đề này có thể mình sẽ nói 1 chút về sự liên hệ giữa các con số và tập hợp. Theo các bạn là nó có liên hệ gì không?




#675485 Một bài tổ hợp từ một bài số học

Đã gửi bởi Karl Heinrich Marx on 27-03-2017 - 23:23 trong Tổ hợp và rời rạc

định nghĩa một xích là $1$ dãy các tập hợp $(A_{a_1};A_{a_2};...;A_{a_t})$ thoả mãn $0<a_1<a_2<...<a_t<T+1$ và $A_{a_i}\subset A_{a_{i+1}} \forall 1\leq i\leq t-1$. Với mỗi tập $A_i$, gọi $f(A_i)$ là xích dài nhất nhận $A_i$ là tập đầu tiên trong xích. Giả sử mọi xích đều có độ dài $\leq n-1$ thì lúc đó tồn tại $n$ tập $A_{b_i} \forall i=\overline{1,n}$ thoả mãn $f(A_{b_i})=f(A_{b_j}) \forall i,j=\overline{1,n}$. Nếu tồn tại $i;j$: $1\leq i< j\leq n$ để $A_{b_i}\subset A_{b_j}$ thì $f(A_{b_i})\geq f(A_{b_j})+1$ (vô lí). Vì vậy $n$ tập trên không thoả mãn đề bài, suy ra giả sử sai. Vậy tồn tại $1$ xích độ dài $n$, nhận $A_t$ làm tập cuối cùng trong dãy. Lúc đó $\left | A_t \right |\geq n\Rightarrow A_t=A$.

Lời giải của bạn chính xác rồi, lời giải của mình hơi khác một tí nhưng chung quy vẫn là nguyên lí Dirichlet, thế theo bạn bài toán này phát biểu một cách số học thì nó như thế nào?




#563984 CMR: $\sum_{k=0}^n {k\choose a}{n-k...

Đã gửi bởi Karl Heinrich Marx on 06-06-2015 - 19:23 trong Tổ hợp và rời rạc

(Nhưng mà phân tích chỗ thừa này ra, lại chính là gợi ý cho lời giải của bài toán này! :)) Thế nên cho phép tôi Stop ở đây!)

 


Ở chỗ này theo thầy, biến đổi thế này
$\sum_{k_1+k_2\le n}{k_1\choose 2}=\sum_{k_1=2}^n\sum_{k_2=0}^{n-k_1}{k_1\choose 2}=\sum_{k=2}^n{k\choose 2}\sum_{k_2=0}^{n-k}1=\sum_{k=2}^n{k\choose 2}(n-k+1)$
Đỡ tốn công giải thích! :D

Mặt khác $\sum_{k=2}^n (n+1-k){k\choose 2}=\sum_{k=2}^n {n+1-k\choose 1}{k\choose 2}=\sum_{k+j=n+1} {j\choose 1}{k\choose 2}={n+2\choose 4}\;\;$ (Áp dụng luôn!)

Cảm ơn thầy đã giúp em chỉnh sửa phần biến đổi gọn gàng hơn!

Ok em đã hiểu vấn đề chỗ này rồi, vì cái sự lặp đó mà mình buộc phải thêm một bạn đánh dấu ở vị trí $k+1$ để dù có chọn $a$ bạn đầu trùng nhau thì vì cái vị trí chốt này mà giúp phân biệt bộ $a$ bạn chọn từ $k$ này khác cũng bộ $a$ bạn đó chọn từ $k$ khác. Thực ra nhìn vào đẳng thức thì cái việc thêm cái bạn ở vị trí chốt này là điều dễ dàng thấy được nhưng em cứ nghĩ chẳng hiểu thêm anh này vào làm gì và thêm một khúc mắc nữa là $a,b$ là những hằng số quên để ý VP làm e tưởng nhầm là tổng vế trái phụ thuộc vào mỗi cặp $a,b$  nên tự hỏi là khi chọn $a+b+1$ bạn bình đẳng như nhau thì tại sao lại chọn bạn chốt ngăn cách là đúng tại giữa $a$ và $b$ người, điều này không tự nhiên. Xem kĩ lại mới thấy thực ra trong từng $a+b+1$ người đấy chọn bất kì ai làm chốt cũng được và biểu thức vế trái chỉ phụ thuộc vào $a+b$ chứ không phụ thuộc vào cặp $(a,b)$.

Bài toán tổng quát thì có vẻ là tương tự. Nếu đây là một kết quả thầy tự mày mò ra thì thật là tuyệt vời! Nhưng kq nhìn đẹp thế này thì chắc chắn là có người tìm ra trước rồi :D Hehe điều đấy không quá quan trọng thầy nhể!




#563904 CMR: $\sum_{k=0}^n {k\choose a}{n-k...

Đã gửi bởi Karl Heinrich Marx on 06-06-2015 - 10:35 trong Tổ hợp và rời rạc

Bằng phương pháp đếm theo hai cách:

CMR: $\sum_{k=0}^n {k\choose a}{n-k\choose b}={n+1\choose a+b+1}$

 

Mở rộng:

 

$\sum_{k_1+k_2+...+k_m=n}{k_1\choose a_1}{k_2\choose a_2}...{k_m\choose a_m}={n+m-1\choose a_1+a_2+...+a_m+m-1}$

Em thấy khá thắc mắc là nếu ta nói số cách chọn $a+b$ bạn trong $n$ bạn là số bằng số cách chọn $a$ bạn từ $k$ bạn đầu tiên và $b$ bạn từ $n-k$ bạn còn lại thì sai cái gì nhỉ? Như vậy thành ra $\sum_{k=0}^n {k\choose a}{n-k\choose b}={n\choose a+b}$?

Thật sự xem những ứng dụng thầy Thanh post ở trên khá hay, một điều thú vị là các số $a_1,a_2,..,a_m$ có thể thay đổi một cách tùy ý miễn đảm bảo giữ nguyên tổng của chúng là được.

Mình có một vài ý thế này:

Nếu thêm một biến $k_{m+1}$ viết lại thế này:

$\sum_{k_1+k_2+...+k_m+k_{m+1}=n}{k_1\choose a_1}{k_2\choose a_2}...{k_m\choose a_m}{k_{m+1}\choose 0}={n+m\choose a_1+a_2+...+a_m+m}$

giờ lược biến $k_{m+1}$ này đi thì ta viết lại thế này:

$\sum_{k_1+k_2+...+k_m \le n}{k_1\choose a_1}{k_2\choose a_2}...{k_m\choose a_m}={n+m\choose a_1+a_2+...+a_m+m}$

Bây giờ cho $m=2,a_1=a_2=1$ thì ta có $\sum_{k_1+k_2 \le n}k_1k_2={n+2\choose 4}$

Hoặc cho $m=2,a_1=2,a_2=0$ thì:

$\sum_{k_1+k_2 \le n}{k_1\choose 2}={n+2\choose 4}$ hay là $\sum_{i=2}^n \sum_{k_1+k_2 =i}{k_1\choose 2}={n+2\choose 4}$

Chú ý là với mỗi số $k$ thì $k$ đóng $n-k+1$ lần vai trò là $k_1$ (với mỗi $i \ge k$ thì $k$ được xét làm $k_1$ một lần). Như vậy ta suy ra:

$\sum_{k=2}^n (n-k+1){k \choose 2} = {n+2\choose 4}$

Dùng kết quả ở trên của thầy Thanh thì ta có:

$(n+1)\sum_{k=2}^n{k \choose 2}=(n+1){n+1 \choose 3}=(n+2){n+1 \choose 3}-{n+1 \choose 3}=4 {n+2\choose 4}-{n+1 \choose 3}$

Vậy $\sum_{k=2}^n k{k \choose 2}=3 {n+2\choose 4}-{n+1 \choose 3}$

Với một đẳng thức linh hoạt thế này có thể điều chỉnh, gán một số giá trị ta sẽ cho ra được thêm nhiều đẳng thức thú vị khác!




#562038 Chứng minh: $\sum_{k=0}^n \left(\frac{-1...

Đã gửi bởi Karl Heinrich Marx on 28-05-2015 - 00:40 trong Tổ hợp và rời rạc

Chứng minh đẳng thức sau:

 

$\sum_{k=0}^n \left(\frac{-1}{2}\right)^k{n\choose k}{2k+1\choose k}= \dfrac{(-1)^n\left(\frac{2n-1-(-1)^n}{2}\right)!!}{\left(\frac{2n+1-(-1)^n}{2}\right)!!}$

 

Trong đó: $\begin{cases}(2m)!!=2^m.m!\\ (2m-1)!!=\dfrac{(2m)!}{2^m.m!}\end{cases}$

Bài này có thể sử dụng kĩ thuật tính hệ số đa thức khá hiệu quả. Tổng cần tính chính là hệ số tự do trong khai triển:

$$ \sum_{k=0}^n \left(\frac{-1}{2}\right)^k{n\choose k} \frac{(x+1)^{2k+1}}{x^k}$$

$$= (x+1) \sum_{k=0}^n {n\choose k}\frac{(-1)^k(x+1)^{2k}}{(2x)^k} = (x+1)\left( 1-\frac{(x+1)^2}{2x} \right)^n=(x+1)\frac{(-1)^n(x^2+1)^n}{2^nx^n}$$

Hệ số tự do của khai triển này chính bằng $$\frac{(-1)^n}{2^n}{n\choose \lfloor \frac{n}{2} \rfloor}$$

Có thể kiểm tra cái này bằng với vế phải, e k biết có phải thầy biến đổi từ giá trị này ra công thức tường minh theo n không có dấu giá trị tuyệt đối không? Nếu là như vậy thì em rất muốn biết thầy biến đổi ntn :D

E không hiểu sao nó k hiện công thức dù đã cố gắng sửa rồi, hi vọng thầy đọc và hiểu được. Em rất thích đọc những bài toán do thầy post vì trước khi post 1 bài toán thầy đều đầu tư ít nhiều vào đó!




#561739 $f(y+f(x))=f(x)f(y)+f(f(x))+f(y)-xy,\;\forall x,y\in...

Đã gửi bởi Karl Heinrich Marx on 26-05-2015 - 20:19 trong Phương trình hàm

Tìm tất cả các hàm số $f:\mathbb{R}\rightarrow \mathbb{R}$ và thoả mãn :

$$f(y+f(x))=f(x)f(y)+f(f(x))+f(y)-xy,\;\forall x,y\in \mathbb{R}$$

Có thể dùng thêm một biến $z$ để linh hoạt hơn,

ta khai triển:

$$f(z+f(y)+f(x))=f(x)f(z+f(y))+f(f(x))+f(z+f(y))-x(z+f(y))=f(x)(f(z)f(y)+f(f(y))+f(z)-zy)+f(f(x))+f(z)f(y)+f(f(y))+f(z)-yz-xz-xf(y)$$

Viết lại thế này cho rõ ràng hơn một chút:

$$f(z+f(y)+f(x))=f(x)f(y)f(z)+(f(f(x))+f(f(y)))+f(z)(f(x)+f(y))+f(z)-z(x+y)+f(x)f(f(y))-zyf(x)-xf(y)$$

Phần đầu mình đã viết ra những hạng tử nhóm lại mà $x$ và $y$ vai trò là như nhau, đổi vai trò $x$ và $y$ ta thu được:

$$ f(x)f(f(y))-zyf(x)-xf(y) = f(y)f(f(x))-zxf(y)-yf(x)$$

Bây giờ xét đến số 0 trước, thì không khó để chứng minh được $f(x)=0$ khi và chỉ khi $x=0$

Giờ xét $x,y$ đều khác 0.

Cho $z=0 \Rightarrow \frac{f(f(x))+x}{f(x)} = const$, cho $z=1 \Rightarrow \frac{f(f(x))}{f(x)} = const$

Vậy $\frac{f(x)}{x} =k$ là hằng số.

Thay vào thì được $k=1$ hoặc $k=-1$




#568017 $7ab+a\mid a^3+(7b+1)^3+7^na$

Đã gửi bởi Karl Heinrich Marx on 25-06-2015 - 07:12 trong Số học

Cho $a,b,n$ là các số nguyên dương thoả mãn :

$$7ab+a\mid a^3+(7b+1)^3+7^na$$

Chứng minh $a$ là lập phương của một số nguyên dương.

Từ đề bài có thể suy ra $a|(7b+1)^3$, do đó với $p$ nguyên tố bất kì mà $p|a \Rightarrow p|7b+1 \Rightarrow p \ne 7$ nên $(p,7^n)=1$

Vì $a|(7b+1)^3$ nên $v_p((7b+1)^3) \ge v_p(a)=v_p(7^na)$ Dễ thấy $v_p(a(7b+1))>v_p(a),v_p(a^3)>v_p(a) \Rightarrow v_p(a)=v_p((7b+1)^3) \vdots 3$

Đây là đpcm.




#566081 CMR: Trong 100 số tự nhiên đó, tồn tại 2 số bằng nhau.

Đã gửi bởi Karl Heinrich Marx on 16-06-2015 - 01:31 trong Bất đẳng thức và cực trị

Gợi ý:

Gợi ý:

Gợi ý:

Haha dĩ nhiên là tìm được chứ thầy, vd lấy 90 số 25 và 10 số 100 thì có tổng bằng 19 rồi :v

Nhưng mà em hiểu ý của thầy, bài toán này câu hỏi đặt ra chỉ là vì mục đích che giấu cái bđt thầy đã nêu ra để đánh đố người khác. Cái này phù hợp vs thi cử :D

Tuy nhiên ta thử đặt ra những vấn đề mới đi vào trọng tâm xem thế nào.

Ví dụ thế này, tìm tất cả số tự nhiên $n$ có thể biểu diễn được dưới dạng

$$\frac{1}{\sqrt{a_1}}+\frac{1}{\sqrt{a_2}}+...+\frac{1}{\sqrt{a_{100}}}$$

trong đó $a_1,a_2,..,a_{100}$ là các số tự nhiên phân biệt.

Hoặc là tìm tất cả các số tự nhiên $n$ mà tồn tại $k$ tự nhiên để

$$n=\frac{1}{\sqrt{a_1}}+\frac{1}{\sqrt{a_2}}+...+\frac{1}{\sqrt{a_k}}$$

trong đó $a_1,a_2,..,a_{k}$ là các số tự nhiên phân biệt.

Khó hơn một chút thì là với mỗi số nguyên dương $t$, hỏi có những số tự nhiên $n$ nào mà thỏa mãn tồn tại $k$ để 

$$\frac{n}{t}=\frac{1}{\sqrt{a_1}}+\frac{1}{\sqrt{a_2}}+...+\frac{1}{\sqrt{a_k}}$$

trong đó $a_1,a_2,..,a_{k}$ là các số tự nhiên phân biệt.

Khó hơn chút nữa thì tìm những số tự nhiên $n$ mà tồn tại $k$ để:

$$\frac{n}{k}=\frac{1}{\sqrt{a_1}}+\frac{1}{\sqrt{a_2}}+...+\frac{1}{\sqrt{a_k}}$$

trong đó $a_1,a_2,..,a_{k}$ là các số tự nhiên phân biệt.

Trong mỗi trường hợp tồn tại trên ta thử tìm số $k$ nhỏ nhất để có thể biểu diễn được xem thế nào.

Hoặc ta cố định $k$, cho trước hai số nguyên dương $k$ và $t$ thì những $n$ nào thỏa mãn:

$$\frac{n}{t}=\frac{1}{\sqrt{a_1}}+\frac{1}{\sqrt{a_2}}+...+\frac{1}{\sqrt{a_k}}$$

trong đó $a_1,a_2,..,a_{k}$ là các số tự nhiên không nhất thiết phân biệt.

 

Haha nếu thầy thấy bài trên có vẻ không thực tế thì giải quyết những vấn đề em nêu trên nhé :D




#564979 $\sum^{995}_{k=0} \dfrac{(-1)^k\...

Đã gửi bởi Karl Heinrich Marx on 11-06-2015 - 18:09 trong Tổ hợp và rời rạc

Ở đây ta thấy rằng: $[x^{2n}]\;(x^2+x+1)^n=1$ và $[x^{2n-1}]\;(x^2+x+1)^n=n$

Như vậy hệ số tự do phải tìm ở TH này là $\frac{-1-n}{2n+1}$

Kết quả cuối cùng lại trở thành:

$\frac{1}{2n+1}-\frac{-1-n}{2n+1}=\frac{2+n}{2n+1}$

 

Vì sao lại thế? Karl Heinrich Marx? Có phải em quên mất trong sigma thiếu cái tổ hợp $ n\choose k$?

Hehe đúng là thiếu cái đấy đấy thầy, lạm dụng quá đúng là chả hay ho gì. Nhưng giật mình là em viết sai cũng chỉ có thầy kiểm chứng và chỉ có em vs thầy trao đổi, dù không hứng thú với cái này đi nữa nhưng chẳng có ai bỏ một chút cố gắng cùng thảo luận với mọi người :|




#563574 $\sum^{995}_{k=0} \dfrac{(-1)^k\...

Đã gửi bởi Karl Heinrich Marx on 04-06-2015 - 23:12 trong Tổ hợp và rời rạc

 

 

Trở lại với tổng:

$\sum_{k=0}^n  \dfrac{(-1)^k{2n+1-k\choose k}}{2n+1-k}=\sum_{k=0}^n \dfrac{(-1)^{n+k}{n+k+1\choose n-k}}{n+k+1}\quad$ (Đơn giản chỉ là đảo thứ tự lấy tổng (reverse index))

$\qquad = \sum_{k=0}^n \dfrac{(-1)^{n+k}(n+k)!}{(2k+1)!(n-k)!}=\dfrac{1}{2n+1}\sum_{k=0}^n \dfrac{(-1)^{n+k}(n+k)!\left[(n+k+1)+(n-k)\right]}{(2k+1)!(n-k)!}$

$\qquad =\dfrac{1}{2n+1}\sum_{k=0}^n \left[(-1)^{n+k}{n+1+k\choose 2k+1}-(-1)^{n-1+k}{n-1+1+k\choose 2k+1}\right]$

$\qquad =\dfrac{1}{2n+1}\left(S_n-S_{n-1}\right)$

 

Với: $S_n=\sum_{k=0}^n (-1)^{n+k}{n+1+k\choose 2k+1}$

 

 

Haha toán học trong em giờ như là một cái gì mờ ảo, thoi thóp rồi thầy ơi :D

Nói một chút về lời giải của thầy Thanh thì thật sự mình thấy ấn tượng với ý tưởng tính theo các dãy $S_n,P_n$, có lẽ vì thầy đã làm việc nhiều với những bài toán thế này nên có thể đưa ra những biến đổi như vậy. Dòng mình bôi đỏ ở trên là một phép biến đổi rất khéo để loại bỏ mẫu số trong sigma. Xin phép dùng ý tưởng này của thầy để tiếp cận một hướng khác. (Như thường lệ lời giải về kiểu này của em vẫn là tính hệ số tự do trong đa thức :D)

Về phương pháp này thì với một sigma chạy từ 0 đến một $n$ (hoặc một hằng số nào đó theo $n$) thì nó vô cùng hiệu quả khi chuyển qua đa thức ta sẽ biến đổi linh hoạt hơn.

Mình xin tiếp tục ở đoạn này:

$$\sum_{k=0}^n  \dfrac{(-1)^k{2n+1-k\choose k}}{2n+1-k}=\qquad =\dfrac{1}{2n+1}\sum_{k=0}^n \left[(-1)^{n+k}{n+1+k\choose 2k+1}-(-1)^{n-1+k}{n-1+1+k\choose 2k+1}\right]$$

Ta sẽ tính riêng rẽ 2 biểu thức này, phần đầu sẽ là hệ số tự do trong biểu thức:

$$ \dfrac{(-1)^n}{2n+1}\sum_{k=0}^n (-1)^k.\frac{(1+x)^{2k+1}}{x^{n+1+k}}=\dfrac{(-1)^n(x+1)}{(2n+1)x^{n+1}}\sum_{k=0}^n (-1)^k.\frac{(1+x)^{2k}}{x^{k}}=\dfrac{(-1)^n(x+1)}{(2n+1)x^{n+1}}\left(1-\frac{(1+x)^2}{x})^n\right)$$

$$=\dfrac{(-1)^n(x+1)(-x^2-x-1)^n}{(2n+1)x^{2n+1}}$$

Rõ ràng hệ số tự do ở đây chính là hệ số của $x^{2n+1}$ trên tử chia cho $2n+1$ và bằng $\frac{1}{2n+1}$.

Tương tự thì biểu thức vế phải là hệ số tự do trong biểu thức:

$$ \dfrac{(-1)^{n-1}}{2n+1}\sum_{k=0}^n (-1)^k.\frac{(1+x)^{2k+1}}{x^{n+k}}= \dfrac{(-1)^n(x+1)}{(2n+1)x^{n}}\sum_{k=0}^n (-1)^k.\frac{(1+x)^{2k}}{x^{k}}=\dfrac{(-1)^{n-1}(x+1)(-x^2-x-1)^n}{(2n+1)x^{2n}}$$

Hệ số tự do trong biểu thức này ta phải tính hệ số của $x^{2k}$ trong biểu thức tử, cái này mình nghĩ tính được. Nhưng ngại tính quá :D




#568024 Tồn tại vô hạn $n$ để $d(n)$ không chia hết $d(a^2+b...

Đã gửi bởi Karl Heinrich Marx on 25-06-2015 - 08:14 trong Số học

Chứng minh rằng với mọi số nguyên dương $k$ luôn tồn tại vô hạn số nguyên dương $n$ sao cho $n$ có đúng $k$ ước nguyên tố và thoả mãn $d(n)$ không chia hết $d(a^2+b^2)$ với mọi cặp số nguyên dương $(a,b)$ thoả mãn $a+b=n$.

Trong đó kí hiệu $d(n)$ là số các ước dương của $n$.

Trước tiên ta nhận xét là chỉ có SCP mới có số ước là lẻ thôi. Vì vậy ta nghĩ đến việc sẽ chọn $n$ là SCP và tìm $n$ sao cho với $a+b=n$ thì $a^2+b^2$ không là SCP. Nếu $a^2+b^2$ là SCP thì phải tồn tại $x,y$ để $a=x^2-y^2,b=2xy$ như vậy $n=x^2-y^2+2xy=(x+y)^2-2xy$

Tuy nhiên đến đây ta thấy là với mọi $n$ chính phương thì phương trình $n=u^2-2v^2$ luôn có nghiệm.

Nhưng không sao ta vẫn có thể có 1 giải pháp khác. Ta chọn $n$ là một SCP mà $k$ ước nguyên tố của $n$ đều có dạng $8t+2$ hoặc $8t+5$, khi đó $2$ không là thặng dư chính phương của những số nguyên tố này nên nếu $u^2-2v^2=n$ thì cả $u$ lần $v$ đều phải chia hết cho tất cả những số nguyên tố này Do vậy dễ thấy $\sqrt{n}$ phải là ước của cả $x+y$ và $y$ suy ra $(x,y)>\sqrt{n}$ như vậy $a^2+b^2=(x^2+y^2)^2 \vdots n^2$ nên dĩ nhiên là $d(n)$ không chia hết cho $d(a^2+b^2)$. Còn nếu $a^2+b^2$ không là SCP thì $2|d(a^2+b^2)$ là $d(n)$ là số lẻ nên cũng không chia hết.

Trong lời giải này vướng mắc một chút ở chỗ với $k$ đủ lớn phải chứng minh tồn tại các số nguyên tố dạng $8t+2$ hoặc $8t+5$. Có nghĩa là phải cm có vô hạn những số nguyên tố dạng này.

Thực ra có một định lí là tồn tại vô hạn số nguyên tố dạng $ak+b$  với $(a,b)=1$ nhưng chứng minh vô cùng khó. Có một đl khác cm đơn giản hơn nhiều là tồn tại vô hạn số nguyên tố dạng $pk+1$. Tuy nhiên cái này k dùng vào bài này. Đây chỉ là một hướng khả thi, có lẽ là có 1 cách tiếp cận khác tốt hơn để tránh vấn đề này.




#568123 Tồn tại vô hạn $n$ để $d(n)$ không chia hết $d(a^2+b...

Đã gửi bởi Karl Heinrich Marx on 25-06-2015 - 16:40 trong Số học

(Cách khác)

Lời giải :

Ta chọn $n$ như sau :

$$n=2^{p-1}p_2p_3...p_k$$

Trong đó $p$ là số nguyên tố, và $p_2,p_3,...,p_k$ là $k$ số nguyên tố phân biệt lớn hơn $3$.

Hiển nhiên có vô số số $n$ như vậy, và hiển nhiên rằng $\omega (n)=k$.

Ta sẽ chứng minh với cách này thì với mọi cặp $(a,b)$ nguyên dương có tổng bằng $n$ thì ta đều có $d(n)\nmid d(a^2+b^2)$.

Thật vậy, giả sử tồn tại một cặp số nguyên dương $(a,b)$ thoả $a+b=n$ và $d(n)\mid d(a^2+b^2)$.

Dễ thấy $d(n)=2^{k-1}.p$. Suy ra :

$$p\mid d(a^2+b^2)\Rightarrow q^{p-1}\mid a^2+b^2$$

với $q$ là một ước nguyên tố nào đó của $a^2+b^2$. Thế thì :

$$q^{p-1}\leq a^2+b^2< (a+b)^2=4^{p}p_2^2p_3^2...p_k^2$$

Rõ ràng nếu $q<4$ thì với $p$ đủ lớn ta sẽ có $q^{p-1}>4^{p-1}p_2^2p_3^2...p_k^2$. Như vậy $q=2,3$.

Nếu $q=3$ thì suy ra $3\mid a^2+b^2$, suy ra $3\mid a,b$. Kéo theo $3\mid n$.

Dễ thấy điều này mâu thuẫn với cách chọn $n$ như trên.

Vậy phải có $q=2$. Dẫn tới :

$$2^{p-1}\mid a^2+b^2$$

Rõ ràng $a,b$ cùng tính chẵn lẻ, nhưng chúng không thể cùng lẻ vì khi đó $a^2+b^2\equiv 2\pmod 4$, vậy phải có $a,b$ cùng chẵn.

Đặt $a=2^A.x,b=2^B.y$ ($x,y$ lẻ), không giảm tổng quát có thể giả sử $A\geq B$.

Ta có :

$$2^{p-1}\mid 2^{2A}x^2+2^{2B}y^2=2^{2B}(2^{2A-2B}x^2+y^2)$$

Chú ý rằng $2^{2A-2B}x^2+y^2\equiv 1,2,3\pmod 4$. Do đó suy ra :

$$2B+1\geq p-1\Rightarrow A\geq B\geq \dfrac{p-1}{2}$$

Suy ra rằng $2^{(p-1)/2}\mid a,b$

Như vậy ta có thể có được biểu diễn sau :

$$a^2+b^2=2^{p-1}(a_1^2+b_1^2)$$

Do $d(n)$ là hàm nhân tính nên :

$$p\mid d(a^2+b^2)=d(2^{p-1}.(a_1^2+b_1^2))=p+d(a_1^2+b_1^2)\Rightarrow p\mid d(a_1^2+b_1^2)$$

Tương tự trên ta được :

$$2^{\frac{p-1}{2}}\mid a_1,b_1$$

Tiếp tục quá trình này, ta sẽ suy ra :

$$2^{\frac{p-1}{2}.N}\mid a,b$$

Với số nguyên dương $N$ tuỳ ý, rõ ràng điều này là vô lí.

Ta có điều cần chứng minh.

Sorry anh hiểu nhầm là $d(a^2+b^2)|d(n)$ thay vì $d(n)|d(a^2+b^2)$, có thể xử lí đoạn sau phần $2^{p-1}|a^2+b^2$ như thế này:

Nếu $v_2(a)>v_2(b) \Rightarrow p-1=v_2(n)=v_2(a+b)=v_2(b) \Rightarrow v_2(a^2+b^2)=2v_2(b)=2p-2 \Rightarrow p \nmid d(a^2+b^2)$

Nếu $v_2(a)=v_2(b)$ thì có thể thấy $v_2(a+b) \ge v_2(a)+1 \Rightarrow v_2(a) \le p-2$. Ngoài ra dĩ nhiên ta có $v_2(a^2+b^2)=2v_2+1<2p-1$ như vậy để $p|d(a^2+b^2$ ta phải có $2v_2(a)+1=p-1$ điều này mâu thuẫn vì một bên chẵn, một bên lẻ.

Thử xem nếu bài toán này mà thay điều kiện $d(n) \nmid d(a^2+b^2)$ thành $d(n) \nmid d(2a^2+2b^2)$ thì phải xử lí thế nào và liệu bài toán còn đúng không?

Ta thử xem với những giá trị $k$ nào (không nhất thiết tổng quát chỉ cần những ước lượng nào đó với $k$) thì thay điều kiện thành $d(n) \nmid d(a^2+kb^2)$ bài toán vẫn đúng.




#564723 $2$ bài toán về con xe

Đã gửi bởi Karl Heinrich Marx on 09-06-2015 - 23:47 trong Tổ hợp và rời rạc

Thực ra 2 bài này em lấy trong 1 quyển sách , cả 2 đều được đánh dấu * và chỉ có phần gợi ý rất ngắn nên em có đăng lên để mọi người thảo luận , giờ thì em đã  lời giải 2 bài này rồi nhưng tại không ai hỏi nữa nên em thôi luôn @@ . Sáng mai em sẽ post ý tưởng 

Anh vẫn có hướng giải hai bài này, anh nghĩ là em post lên vì chưa giải được vì vậy anh chỉ muốn nói là nếu chưa giải được em hãy post lên ý tưởng của em, mọi người cùng thảo luận thì nhìn ra tại sao mình không giải được, sẽ thấy được ưu nhược điểm trong cách tiếp cận của mình và hơn nữa như vậy mới dễ có bạn vào tham gia thảo luận với em, biết đâu tìm được những ý tưởng khác hay hơn. Anh sẽ nói sơ ý tưởng của anh trong 2 bài này, chỉ là một chút ý tưởng anh cho là khả thi, cũng chưa thử cụ thể là có làm được hay không.

Bài đầu thì để ý một chút tính chất là nếu đổi vị trí các cột với nhau và đổi vị trí các hàng với nhau thì chẳng ảnh hưởng gì đến bài toán nên nếu như ta tìm được một cặp khác màu đặt 2 con xe lên đó, dùng phép đổi cột và đổi hàng ta có thể đưa 2 ô vừa đặt quân nằm trên hv 2x2 của góc bàn cờ, đến đây dùng một chút lập luận về số màu để đưa về hv $(n-2)$x$(n-2)$. Đặt $n=2k$ chứng minh bài toán đúng với $k=2$ sau đó có thể quy nạp theo $k$.

Bài 2 thì chú ý là một hàng có nhiều nhất $2$ ô được chọn và một cột cũng thế và dùng tính chất một hàng mà chứa 2 ô được chọn thì 2 ô đó chiếm 2 cột chứa 1 ô, ngược lại một cột chứa ô được chọn thì 2 ô đó chiếm 2 hàng chứa 1 ô, kết quả sẽ là $4n$.




#564647 $2$ bài toán về con xe

Đã gửi bởi Karl Heinrich Marx on 09-06-2015 - 19:07 trong Tổ hợp và rời rạc

Hungari 81 Các ô vuông của bàn cờ kích thước $n\times n$ ( trong đó $n$ là số chẵn lớn hơn 2 ) , được tô bằng $\frac{n^{2}}{2}$ màu sao cho mỗi màu tô đúng 2 ô . Chứng minh rằng trên bàn cờ có thể đặt $n$ con xe sao cho chúng đứng trên các ô vuông có màu khác nhau và chúng không "ăn" được nhau   

Nam Tư 75Số lớn nhất các con xe có thể đặt trên bàn có kích thước $3n\times 3n$ có thể là bao nhiêu , để sao cho mỗi con xe chỉ bị "ăn" không nhiều hơn 1 con khác trong số các con xe còn lại

Khi post 2 bài này em đã suy nghĩ chưa, nếu chưa giải ra thì em đã có ý tưởng hướng đi nào mà chưa ra được kq chưa, cùng đưa lên mọi người sẽ thảo luận với em.




#564652 Tìm số tập con của tập $S=\begin{Bmatrix} 1;2;...;n...

Đã gửi bởi Karl Heinrich Marx on 09-06-2015 - 19:34 trong Tổ hợp và rời rạc

Có 2 ý tưởng để đếm bài này, đầu tiên dễ thấy là truy hồi. Gọi $S_n$ là tập hợp các tập con thỏa mãn với $n$ thì một tập mà thuộc $S_{n+1}$ nó sẽ là một tập trong $S_n$ còn nếu không thì nó chứa $n+1$ suy ra các phần tử của nó bé hơn hoặc bằng $n-2$ mà thỏa mãn đk bài toán. Vậy $|S_{n+1}|=|S_{n-2}|+|S_n|$.

Ý tưởng thứ hai là lập một song ánh để cố phá cái điều kiện hơn kém ít nhất 3 đơn vị đi. Bây giờ giả sử một tập hợp thỏa mãn đề bài mà có $i$ phần tử là $a_1<a_2<...<a_i$ khi đấy nếu ta chuyển sang cái tập này ${b_j=2(i-j)+a_j, 1 \le j \le i}$ thì đây là một tập con bình thường của tập $S$ mà trong đó các phần tử của nó đều không nhỏ hơn $2i-1$. Ngược lại từ một tập con của $n$ có $i$ phần tử thỏa mãn $b_1<b_2<..<b_i$ và $b_1 \ge 2i-1$ khi đó lật lại ta có tập ${a_j=b_j-2(i-j), 1 \le j \le i}$ đây là một tập con thỏa mãn đề bài, vậy số tập con có $i$ phần tử thỏa mãn đề bài là ${n-2i+2\choose i}$. Cho $i$ chạy từ $0$ đến $n$ tính ra kết quả.

Kể ra ta thu được một đẳng thức tổ hợp bằng 2 cách đếm.




#564726 Tìm số tập con của tập $S=\begin{Bmatrix} 1;2;...;n...

Đã gửi bởi Karl Heinrich Marx on 10-06-2015 - 00:03 trong Tổ hợp và rời rạc

Em cứ nghĩ là sẽ tính được để cho thêm thầy Thanh một đẳng thức vào bộ sưu tập chứ :D




#641838 Bài tổ hợp BMO 2010

Đã gửi bởi Karl Heinrich Marx on 23-06-2016 - 07:03 trong Tổ hợp và rời rạc

Ta sẽ chứng minh là không phải luôn luôn sắp được như vậy. Ta thiết kế một mối quan hệ giữa các đứa trẻ chỉ ra nó không thỏa mãn. Đầu tiên xét một đồ thị hình cây với đỉnh là một đứa trẻ (bậc 0). Đứa trẻ này có 3 người quen (bậc 1), 3 đứa này mỗi đứa lại quen 3 đứa khác nữa (bậc 2),.... cứ vậy mỗi đứa trẻ xuất hiện trong đồ thị lại quen với 3 đứa trẻ khác. Ta thành lập được một cây tam phân $n$ bậc, ta có thể cho $n=2010$ cũng được.

Giả sử tồn tại cách sắp xếp để thỏa mãn đk đề bài với những đứa trẻ thuộc cái cây này.

Xét vị trí của đứa trẻ ở đỉnh bậc 0 là $a$. Lấy vị trí của mỗi đứa trẻ trừ cho $a$, ta xem như vị trí của đứa trẻ ở bậc 0 là 0. Khi đó ta có thể chứng minh quy nạp theo $k$ là nếu một đứa trẻ thuộc cây tam phân ở bậc không vượt quá $k$ thì trị tuyệt đối vị trí của nó không vượt quá $2011k$.

Tuy nhiên tổng số đứa trẻ thuộc cây tam phân mà có bậc không vượt quá $k$ là $\frac{3^{k+1}+1}{2}$.

Ta chọn được $k$ đủ lớn mà bé hơn $2010$ để $\frac{3^{k+1}+1}{2}>2.2011k+1$.

Điều này chứng tỏ phải có 2 em trùng vị trí, vô lí.

Có thể tối ưu số em bé là $\frac{3^{k+1}+1}{2}$ với $k$ thỏa mãn bđt trên.




#566327 Tìm nghiệm nguyên

Đã gửi bởi Karl Heinrich Marx on 17-06-2015 - 05:39 trong Số học

Tìm nghiệm nguyên của phương trình:

(x+y2)(x2+y)=(y-x)2

Trước tiên đặt $d>0$ là UCLN của $x$ và $y$ trong đó $x=da,y=db \Rightarrow (a,b)=1$ như vậy pt trên tương đương với:

$$(a+db^2)(b+da^2)=(a-b)^2 (*)$$

Không mất tính tổng quát ta giả sử $|a| \ge |b|$, từ biểu thức trên ta suy ra:

$$4a^2 \ge (a-b)^2 \ge da^2+b \ge 0 \Rightarrow d \le 5$$

Nếu mà $d=5$ thì những đẳng thức trên xảy ra đồng thời và ta có $a=-b$

Khi đó thay vào $(*)$ ta được:

$$(5b-1)(5b+1)=4 \Rightarrow 5b^2=1$$

loại nghiệm này.

Vậy $1 \le d \le 4$

Ngoài ra ta biến đổi biểu thức ban đầu theo một cách khác:

$(x+y^2)(y+x^2)=(y-x)^2 \Leftrightarrow xy(xy+2)=(1-x-y)(x^2-xy+y^2) \Leftrightarrow ab(d^2ab+2)=(1-da-db)(a^2-ab+b^2)$

Vì $a,b$ nguyên tố cùng nhau nên biểu thức $a^2-ab+b^2$ luôn là số lẻ. Do đó nếu $d$ chẵn thì bên trái chia hết cho 2 mà bên phải thì không. Vậy $d$ phải lẻ.

Ngoài ra cũng từ biến đổi cuối cùng ở trên ta suy ra được $(a^2-ab+b^2)|d^2ab+2$.

Nếu $d=1$ thì ta suy ra $ab+2 \ge a^2-ab+b^2 \Rightarrow 2 \ge (a-b)^2$

Mà $a$ không thể bằng $b$ nên ta suy ra $|a-b|=1$

Đến đây thay vào giải khá đơn giản ra $a=1,b=0$ và ngược lại.

Bây giờ nếu $d=3$ cũng từ biểu thức trên ta suy ra $ab|(1-da-db) \Rightarrow a|1-3b$

Đặt $|1-3b|=k|a|$ thì ta chặn được $1 \le k \le 2$

Đến đây thì còn vài trường hợp thay vào và giải ra đáp án, mình không rõ là có cách nào để loại bớt trường hợp nữa không nhưng đến đây chắc khả thi rồi.




#563156 Chứng minh rằng tồn tại 2 nhà phê bình bỏ phiếu như nhau

Đã gửi bởi Karl Heinrich Marx on 03-06-2015 - 06:14 trong Tổ hợp và rời rạc

 

Có 3366 nhà phê bình điện ảnh bỏ phiếu cho giải Oscar. Mỗi một nhà phê bình bỏ phiếu cho đúng 1 nam diễn viên và đúng 1 nữ diễn viên. Sau khi các nhà phê bình bỏ phiếu xong, người ta thấy rằng với mỗi n thuộc {1, 2, ..., 100} đều có một nam diễn viên hoặc 1 nữ diễn viên được đúng n phiếu. Chứng minh rằng tồn tại 2 nhà phê bình bỏ phiếu như nhau (tức là bỏ phiếu cho cùng 1 nam diễn viên và 1 nữ diễn viên).

(Balkan MO 2015)

 

Trước tiên ta khai thác một số điều từ giả thiết. Tổng số phiếu sẽ là $\frac{100.101}{2}=5050$, mà chỉ có $3366$ nhà phê bình nên phải có ít nhất $5050-3366=1684$ người bỏ 2 phiếu. Rõ ràng từ câu hỏi của bài toán thì ta phải xem xét đến $1684$ ông này, để chứng minh có $2$ ông bỏ phiếu giống nhau thì chỉ có thể rơi vào $1684$ vị này là khả thi thôi. Tức là nếu xét các cặp được cùng một nhà phê bình bỏ phiếu thì trong $100$ diễn viên đề cập trong đề phải có ít nhất $1684$ cặp nam nữ. Đến đây ý tưởng sẽ phải chứng minh không thể có đến $1684$ cặp được. Bây giờ ta sẽ cố gắng tạo ra nhiều cặp nam nữ nhất trong số $100$ người này

Không mất tính tổng quát giả sử có $k$ người nam và $100-k$ người nữ trong đó $k \le 50$. Như vậy nhiều nhất ta tạo ra được $(100-k)k$ cặp. Tuy nhiên như thế thì $k$ nam sẽ có $100-k$ phiếu và $100-k$ nữ sẽ có $k$ phiếu. Như vậy số phiếu của những người này đều không nhỏ hơn $k$, điều này không hợp lí. Buộc ta phải bỏ bớt các cặp trong này để đưa một số giá trị về $1,2,3,..,k-1$, chú ý là $100-k \ge k$ nên để loại bỏ ít cặp nhất ta sẽ chọn ra $k-1$ cô gái trong $100-k$ người có $k$ phiếu và bỏ bớt phiếu những người này tạo thành dãy $1,2,..,k-1$. Như vậy ta đã bỏ đi $k.(k-1)-(1+2+..+k-1)=\frac{k(k-1)}{2}$. Vậy chỉ có thể tạo ra nhiều nhất là $(100-k)k-\frac{k(k-1)}{2}=k(100-k-\frac{k-1}{2})$. Áp dụng Cauchy thì ta có:

$$ \frac{3}{2}k(100-k-\frac{k-1}{2}) \le \frac{1}{4}(100+1/2)^2<\frac{100.101}{4} \Rightarrow k(100-k-\frac{k-1}{2}) < \frac{100.101}{6}=\frac{5050}{3}<1684$$

Vậy là kết quả được chứng minh.

Dù là giải đi đến kết quả thậm chí có thể thấy là ta hoàn toàn có thể tổng quát $100$ thành $n$ và thay đổi số nhà phê bình bé hơn $\frac{n(n+1)}{3}$. Tuy nhiên chúng ta cần nhìn nhận vấn đề cẩn thận hơn một chút, việc chứng minh này chưa có nhiều ý nghĩa lắm. Mình sẽ nêu ra một số vấn đề để các bạn tìm hiểu để hiểu rõ bài toán hơn:

1. Đầu tiên hãy xem khi tiếp cận bài toán các bạn suy nghĩ ntn? Dĩ nhiên các bạn sẽ phải hình dung đến một cấu hình tốt nhất nào đấy để cố gắng tìm điểm mâu thuẫn ở đó.

2.. Nếu ta tăng số nhà phê bình thêm 1 hoặc 2 đơn vị. Khi đó đpcm không đúng nữa, vậy cấu hình giám khảo chấm như thế nào để nó không đúng (đây có thể gọi là cấu hình "tốt nhất", dĩ nhiên hướng xây dựng cấu hình này sẽ dựa vào dấu bất đẳng thức ở trên). Từ đó các bạn có thể thấy được cách đưa bài toán vào trạng thái tối ưu.

3. Có một vấn đề là ta mới chỉ ra mâu thuẫn ở việc đánh giá của 1684 giám khảo chấm cả 2 phiếu trên 100 người thôi. Giả sử ta đáp ứng được số cặp trong 100 người thỏa mãn bđt trên, liệu những người chỉ bỏ 1 phiếu còn lại có thể hoàn thành được đk của bài toán là cứ $1 \le i \le 100$ thì có một người có $i$ phiếu? Thay $100$ bằng $n$ thì sẽ rơi vào những vấn đề gì? Đổi lại tìm số giám khảo nhiều nhất để đảm bảo có 2 người bỏ phiếu giống nhau thì xử lí thế nào với $n$?

4. Nếu vấn đề đòi hỏi đến 2 cặp giám khảo bỏ phiếu giống nhau thì thế nào? hoặc một giám khảo có thể bỏ phiếu cho 2 người nam và 1 người nữ thì ntn?

Khi tìm hiểu bài toán đủ nhiều các bạn có thể tự đặt ra thêm những câu hỏi và tự giải quyết chúng. Đôi khi có thể đặt ra rồi không giải quyết được nhưng chắc chắn là không vô ích!

Hi vọng các bạn tham gia thảo luận nhiều hơn là đưa ra một lời giải.

P/s em chủ topic: À muốn hỏi e một vấn đề nhỏ, thầy Thông trong chữ kí của em có phải là thầy Nguyễn Văn Thông ở LQĐ Đà Nẵng k nhỉ?




#563532 Chứng minh rằng tồn tại 2 nhà phê bình bỏ phiếu như nhau

Đã gửi bởi Karl Heinrich Marx on 04-06-2015 - 21:58 trong Tổ hợp và rời rạc

dạ đúng rồi anh :))

Haha anh chỉ hỏi vậy thôi chứ nghĩ chắc 90% rồi, mấy ai tên Thông mà bá đạo hơn thầy :v

Anh nghĩ khi post bài toán lên em cũng nên tham gia thảo luận nhiều hơn để các bạn khác cùng tham gia thảo luận chứ, dù sao cũng nên nêu ra những cảm nhận của mình về lời giải, những hướng suy nghĩ của em hoặc những cái gì em cho là có thể khai thác thêm từ bài toán, hoặc có thể giải quyết những câu hỏi anh nêu ở trên. Nên thảo luận nhiều hơn là đưa 1 đề toán và post 1 lời giải em ạ!